Đến nội dung

nguyentrungphuc26041999 nội dung

Có 382 mục bởi nguyentrungphuc26041999 (Tìm giới hạn từ 28-04-2020)



Sắp theo                Sắp xếp  

#495991 CMR: $\sum \frac{a^{2}+bc}{\sqrt...

Đã gửi bởi nguyentrungphuc26041999 on 29-04-2014 - 22:07 trong Bất đẳng thức - Cực trị

đổi biến cho dễ nhìn thay$\sqrt{a}=a,\sqrt{b}=b,\sqrt{c}=c$

$\Rightarrow a+b+c=1$

cần chứng minh $A=\sum \frac{a^{4}+b^{2}c^{2}}{a^{2}\sqrt{2\left ( b^{2}+c^{2} \right )}}\geq 1$

$A=\sum \frac{a^{2}}{\sqrt{2\left ( b^{2}+c^{2} \right )}}+\sum \frac{\frac{b^{2}c^{2}}{a^{2}}}{\sqrt{2\left ( b^{2}+c^{2} \right )}}\geq 1$

xét $B=\sum \frac{a^{2}}{\sqrt{\left ( b^{2}+c^{2} \right )}}$

đặt $\sqrt{a^{2}+b^{2}}=x,\sqrt{b^{2}+c^{2}}=y,\sqrt{c^{2}+a^{2}}=z$

$B=\sum \frac{y^{2}+z^{2}-x^{2}}{2x}= \sum \frac{y^{2}}{2x}+\sum \frac{z^{2}}{x}-x-y-z\geq \frac{x+y+z}{2}=\frac{\sum \sqrt{a^{2}+b^{2}}}{2}\geq \sum \frac{a+b}{2\sqrt{2}}=\frac{a+b+c}{\sqrt{2}}$

$\Rightarrow \frac{a^{2}}{\sqrt{2\left ( b^{2}+c^{2} \right )}}\geq \frac{a+b+c}{2}=\frac{1}{2}$ $\left ( 1 \right )$

xét $C= \frac{\frac{b^{2}c^{2}}{a^{2}}}{\sqrt{2\left ( b^{2}+c^{2} \right )}}\geq \frac{\left ( \sum \frac{bc}{a} \right )^{2}}{\sum \sqrt{2\left ( b^{2}+c^{2} \right )}}$

ta chứng minh $\left ( \sum \frac{ab}{c} \right )\geq 3\sum a^{2}$

$\Leftrightarrow \sum \frac{a^{2}b^{2}}{c^{2}}+2\sum a^{2}\geq 3\sum a^{2}$ (đúng)

$\Rightarrow C\geq \frac{3\left ( a^{2}+b^{2}+c^{2} \right )}{\sqrt{2\left ( b^{2}+c^{2} \right )}}$

$\Rightarrow C\geq \frac{\sum a^{2}}{\sqrt{\frac{2}{9}\left ( b^{2}+c^{2} \right )}}\geq \frac{2\sum a^{2}}{2\sum a^{2}+\frac{2}{3}}=\frac{2}{2+\frac{2}{3\sum a^{2}}}\geq \frac{2}{2+\frac{2}{\left ( a+b+c \right )^{2}}}=\frac{1}{2}$ $\left ( 2 \right )$

 

từ $\left ( 1 \right ),\left ( 2 \right )$ suy ra dpcm




#495978 $A=x^3+y^3+x^2+y^2+5(x+y)+\frac{1}{x}+\fra...

Đã gửi bởi nguyentrungphuc26041999 on 29-04-2014 - 21:10 trong Bất đẳng thức và cực trị

Cái này liệu có trái dấu không bạn ? 

không

$5\left ( x+y \right )+\frac{4}{x+y}=\frac{19\left ( x+y \right )}{4}+\frac{x+y}{4}+\frac{4}{x+y}\geq \frac{19\left ( x+y \right )}{4}+2$




#495972 $A=x^3+y^3+x^2+y^2+5(x+y)+\frac{1}{x}+\fra...

Đã gửi bởi nguyentrungphuc26041999 on 29-04-2014 - 20:54 trong Bất đẳng thức và cực trị

Cho x,y>0 và x+y+xy=8. tìm GTNN $A=x^3+y^3+x^2+y^2+5(x+y)+\frac{1}{x}+\frac{1}{y}$

Từ giả thiết ta có 

$\frac{\left ( x+y \right )^{2}}{4}+x+y\geq 8$

$\Rightarrow x+y\geq 4$

ta có $A\geq \frac{\left ( x+y \right )^{3}}{4}+\frac{\left ( x+y \right )^{2}}{2}+5\left ( x+y \right )+\frac{4}{x+y}$

dễ rồi




#493577 Tìm GTLN của: $P=xy+yz+zx$

Đã gửi bởi nguyentrungphuc26041999 on 17-04-2014 - 20:49 trong Bất đẳng thức và cực trị

Cho các số thực dương thoả: $(x+y)(y+z)(z+x)=8$. Tìm GTLN của:

$P=xy+yz+zx$

$8=\prod \left ( x+y \right )\leq \frac{\left ( 2x+2y+2z \right )^{3}}{27}$

$\Rightarrow x+y+z\geq 3$

sử dụng bất đẳng thức phụ $8=\prod \left ( x+y \right )\geq \frac{8}{9}\left ( xy+yz+zx \right )\left ( x+y+z \right )$

$\Rightarrow xy+yz+xz\leq \frac{9}{x+y+z}\leq 3$

dấu bằng xảy ra khi $x=y=z=1$




#492246 Trận 7 - PT, HPT đại số

Đã gửi bởi nguyentrungphuc26041999 on 11-04-2014 - 20:42 trong Thi giải toán Marathon cấp THCS 2014

Giải phương trình: $2x^{2}+5x-1=7\sqrt{x^{3}-1}$

Đề thi của l4lzTeoz

Ta có 

$2x^{2}+5x-1=7\sqrt{x^{3}-1}$ ( điều kiện xác định $x\geq 1$)

$\Leftrightarrow 2x^{2}+5x-1=7\sqrt{\left ( x-1 \right )\left ( x^{2}+x+1 \right )}$

đặt $\left\{\begin{matrix} \sqrt{x^{2}+x+1}=a & \\ \sqrt{x-1}=b & \end{matrix}\right.$

$\Rightarrow 2a^{2}+3b^{2}=2x^{2}+5x-1$

$\Leftrightarrow 2a^{2}+3b^{2}=7ab$

$\Leftrightarrow 2a^{2}-7ab+b^{2}=0$

$\left ( 2a-b \right )\left ( a-3b \right )=0$

$\Leftrightarrow \left\{\begin{matrix} 2a=b & \\ a=3b & \end{matrix}\right.$

nếu $2a=b$

$\Leftrightarrow 2\sqrt{x^{2}+x+1}=\sqrt{x-1}$

$\Leftrightarrow 4x^{2}+4x+4=x-1$

$\Leftrightarrow 4x^{2}+3x+5=0$ (vô nghiệm do $x\geq 1$

nếu $a=3b$

$\Leftrightarrow \sqrt{x^{2}+x+1}=3\sqrt{x-1}$

$\Leftrightarrow x^{2}+x+1=9x-9$

$x^{2}-8x+10=0$

$\Delta '=4^{2}-10=6$

phương trình có 2 nghiệm $\left\{\begin{matrix} x=4+\sqrt{6}\left ( TMDK \right ) & \\ x=4-\sqrt{6} & \left ( TMDK \right ) \end{matrix}\right.$

vậy phương trình có 2 nghiệm $x=\left \{ 4+\sqrt{6},4-\sqrt{6} \right \}$

 

 

  d =10

  S =17+10.3=47




#490760 Tìm $Min_{P}=\dfrac{x}{y^2}+\dfr...

Đã gửi bởi nguyentrungphuc26041999 on 05-04-2014 - 15:37 trong Bất đẳng thức và cực trị

Cho $x,\,y,\,z>0$ thỏa $xyz=1.$ Tìm giá trị nhỏ nhất của biểu thức:$$P=\dfrac{x}{y^2}+\dfrac{y}{z^2}+\dfrac{z}{x^2}+\dfrac{9}{x+y+z}$$

đặt$x=\frac{a}{c},y=\frac{b}{a},z=\frac{c}{b}$

ta tìm min của $A=\sum \frac{a^{3}}{bc^{2}}+\frac{9abc}{a^{2}c+b^{2}a+c^{2}b}$

 

theo bất đẳng thức BCS $A=\sum \frac{a^{4}}{abc^{2}}+\frac{9abc}{a^{2}c+b^{2}a+c^{2}b}\geq \frac{\left ( a^{2}+b^{2}+c^{2} \right )^{2}}{abc\left ( a+b+c \right )}+\frac{9abc}{a^{2}c+b^{2}a+c^{2}b}$

$\frac{\left ( a^{2}+b^{2}+c^{2} \right )^{2}}{abc\left ( a+b+c \right )}+\frac{9abc}{a^{2}c+b^{2}a+c^{2}b}\geq 2\sqrt{\frac{9\left ( a^{2}+b^{2}+c^{2} \right )^{2}}{\left ( a+b+c \right )\left ( a^{2}c+b^{2}a+c^{2}b \right )}}$

đến đây ta chứng minh $\left ( a^{2}+b^{2}+c^{2} \right )^{2}\geq \left ( a+b+c \right )\left ( a^{2}c+b^{2}a+c^{2}b \right )$

nhân toe ra là xong




#489854 Cho $a;b>0$ thoả: $a+b=4\sqrt{ab}$. Tí...

Đã gửi bởi nguyentrungphuc26041999 on 31-03-2014 - 19:53 trong Đại số

1.Cho $a;b>0$ thoả: $a+b=4\sqrt{ab}$. Tính $\frac{a}{b}$

2.Một công việc cần 420 ngày công được thi công bởi một nhóm công nhân. Tìm số công nhân của nhóm này, biết nếu nhóm được tăng cường thêm 5 công nhân thì họ hoàn toàn thành công việc trước 7 ngày.

1.Chia cả 2 vế cho b

$\frac{a}{b}+1=4\sqrt{\frac{a}{b}}$

bây giờ tìm $\sqrt{\frac{a}{b}}$ bằng cách giải phương trình bậc 2 là xong




#489712 Trận 6 - Phương trình nghiệm nguyên, đồng dư, chia hết

Đã gửi bởi nguyentrungphuc26041999 on 30-03-2014 - 20:24 trong Thi giải toán Marathon cấp THCS 2014

ta có ,với $x=1$

$\sqrt{2025x^{2}+2012x+3188}=\sqrt{2025+2012+3188}=85$

$\Rightarrow 85=2013-2011y+2094$

$\Leftrightarrow 4022=2011y$

$\Leftrightarrow y=2$ (thoả mãn $y$ là số tự nhiên)

Cặp số nguyên $\left ( x,y \right )=\left ( 1,2 \right )$ thoả mãn phương trình.

Vậy luôn tồn tại cặp số nguyên $\left ( x,y \right )$ thoả mãn phương trình

 

  d =9

 S =17+9x3 = 44




#488353 Cho tam giác $ABC$ có $\angle BAC=60^{\circ...

Đã gửi bởi nguyentrungphuc26041999 on 23-03-2014 - 09:42 trong Hình học

Cho tam giác $ABC$ có $\angle BAC=60^{\circ}$. $H;I;O$ lần lượt là trực tâm; tâm đường tròn nội tiếp và ngoại tiếp của tam giác $ABC$. Chọn ý đúng nhất :
a. $H;I;O$ thẳng hàng
b. $B;C;H;I;O$ cùng thuộc 1 đường tròn

Và chứng minh ý bạn chọn !

Tất nhiên là B

$\angle BHC=\angle BIC=\angle BOC=120$




#488352 $4(\frac{1}{a}+\frac{1}{b...

Đã gửi bởi nguyentrungphuc26041999 on 23-03-2014 - 09:39 trong Bất đẳng thức và cực trị

Lỗi $\LaTeX$ nhiều quá!

Chả hiểu phần lỗi bạn viết gì! mình giải luôn phần sau vậy nhé! 

Ta có $a^{3}+ b^{3}+ c^{3}-3abc=\left ( a+b+c \right )\left ( a^{2} +b^{2}+c^{2}-ab-ac-bc\right )\Rightarrow 3=3abc+(a+b+c)(a^{2}+b^{2}+c^{2}-ab-ac-bc)$

Mà $a^{2}+b^{2}+c^{2}\geq ab+ac+bc$

Suy ra $3abc\geq 3\Rightarrow abc\geq 1$

Từ đó suy ra $4(\frac{1}{a}+\frac{1}{b}+\frac{1}{c})+5(a^2+b^2+c^2)\geq 4(\frac{9}{a+b+c})+5.\frac{\left ( a+b+c \right )^{2}}{3}=\frac{9}{a+b+c}+\frac{9}{a+b+c}+\frac{9}{a+b+c}+\frac{9}{a+b+c}+\frac{\left ( a+b+c \right )^{2}}{3}+\frac{\left ( a+b+c \right )^{2}}{3}+\frac{\left ( a+b+c \right )^{2}}{3}+\frac{\left ( a+b+c \right )^{2}}{3}+\frac{\left ( a+b+c \right )^{2}}{3}\geq 9\sqrt[9]{\frac{\left ( a+b+c \right )^{8}.9^{4}}{3^{5}}} \geq 9\sqrt[9]{3^{8}.3}=27\rightarrow Q.E.D$

Chỗ này có vấn đề rồi phải là $abc\leq 1$




#487261 Trận 5 - toán rời rạc

Đã gửi bởi nguyentrungphuc26041999 on 16-03-2014 - 20:38 trong Thi giải toán Marathon cấp THCS 2014

Cho bàn cờ vua $8 \times 8$. Theo thứ tự từ trái qua phải, từ trên xuống dưới, ta làm việc sau:

Trong ô cờ thứ nhất đặt 1 hạt ngô

Trong ô cờ thứ hai đặt 2 hạt ngô

Trong ô cờ thứ ba đặt 4 hạt ngô

...

Trong ô cờ thứ 64 đặt $2^{63}$ hạt ngô.

 

Một con mã ô đầu tiên của bàn cờ, nó đi lòng vòng và ăn các hạt ngô trong ô nó nhảy đến( con mã di chuyển theo hình chữ L - 3 ô như đối với môn cờ vua) nhưng nó không ăn ở ô đầu tiên và không nhảy trở lại ô đầu tiên. Sau mỗi lần nó ăn người ta lại đặt số ngô bằng số ngô ban đầu vào trong ô đó. Sau khi con mã đi xong nó quay trở về ô đầu tiên và ăn nốt hạt ngô ở ô đó.

 

Hãy CM rằng số ngô mà con mã ăn chia hết cho 3.

 

Ta có 

$2^{2a}=4^{k}\equiv 1^{a}\left ( mod 3 \right )$

suy ra $2^{a}$ chia $3$ dư $1$

$2^{2b+1}=2.4^{k}\equiv 2.1^{b}\left ( mod3 \right )$

$2^{2b+1}$ chia $3$ dư $2$

Quay trở lại bài toán,đánh thứ tự cột từ trái sang phải là $a,b,c,d,e,f,g,h$,đánh thứ tự cột là $1,2,3,4,5,6,7,8$

gọi ô có số hạt ngô khi chia cho $3$ dư $1$ là ô trắng,gọi ô có số hạt ngô khi chia cho $3$ dư $2$ là ô đen

dễ thấy ô $a1$ là ô trắng,trong mỗi hàng, ô đen và ô trắng xen kẽ nhau theo nên ô cuối của hàng 1 (ô $a8$) là ô đen do mỗi hàng có 8 ô.

vì thế ô cuối cùng của hàng $2$ là ô trắng vì thế ô đầu tiên của hàng 2 là ô đen do mỗi hàng có 8 ô.

Lập luận như thế ta cũng có ở cột $a$ ô trắng và ô đen xen kẽ nhau.Nhưng do ô đen và ô trắng mỗi hàng xen kẽ nhau nên ô đen và ô trắng ở mỗi hàng cũng xen kẽ nhau.

Vậy ô trắng và ô đen đều xen kẽ nhau ở mỗi hàng và mỗi cột ( theo bàn cờ vua )

Dễ thấy sau mỗi lượt đi,con mã nhảy từ trắng sang ô đen và ô đen sang ô trắng.

Vì thế,sau mỗi 2 lần đi,con mã sẽ nhảy quá lanf ô trắng và 1 lần ô đen,số hạt ngô con ngựa ăn được sẽ chia hết cho 3(cho dù chúng có nhảy qua ô chúng đã đi qua hay không vì sau mỗi lần nó ăn,người ta lại đặt lại số ngô nó ăn)do $2^{2a}+2^{2b+1}\equiv 1+2\left ( mod 3 \right )$ hay chia hết cho $3$ 

Ta có, con ngựa xuất phát ở ô trắng và kết thúc ở ô trắng nên số nước đi của chúng là lẻ nhưng do ở ô đầu tiên nó không ăn hạt ngô đó,vì vậy số nước đi của nó là chẵn,nên số hạt ngô nó ăn chia hết cho 3 (đpcm)




#486059 a.$\left\{\begin{matrix} \frac{1...

Đã gửi bởi nguyentrungphuc26041999 on 06-03-2014 - 19:56 trong Phương trình, hệ phương trình và bất phương trình

Giải hệ phương trình:

a. $\left\{\begin{matrix} \frac{1}{x}+\frac{1}{y}+\frac{1}{z}=2\\ \frac{2}{xy}-\frac{1}{z^{2}}=4 \end{matrix}\right.$

Câu 1 thì đặt cho dễ bạn tự đặt nhé

bình phương vế 1 lên ta được

$a^{2}+b^{2}+c^{2}+2\left ( ab+bc+ca \right )= 2ab-c^{2}$

$a^{2}+b^{2}+2c^{2}= 2\left ( ac+bc \right )$

cái này dùng bất đẳng thức là ra 




#485216 Trận 4 - Bất đẳng thức

Đã gửi bởi nguyentrungphuc26041999 on 28-02-2014 - 22:16 trong Thi giải toán Marathon cấp THCS 2014

Ta có 

$x^{2}+y^{2}\geq 2xy$ (mọi $x,y\in R$)

$\Rightarrow \left ( x+y \right )^{2}\geq 4xy$

$\Rightarrow \left ( x+y \right )^{3}+\left ( x+y \right )^{2}\geq \left ( x+y \right )^{3}+4xy\geq 2$

$\Rightarrow \left ( x+y \right )^{3}+\left ( x+y \right )^{2}\geq 2$

$\Rightarrow \left ( x+y-1 \right )\left [ \left ( x+y \right )^{2}+2\left ( x+y \right )+2 \right ]\geq 0$

Do $\left ( x+y \right )^{2}+2\left ( x+y \right )+2= \left ( x+y+1 \right )^{2}+1>0$

$\Rightarrow x+y-1 \geq 0$

$\Rightarrow x+y \geq 1$

Theo bất đăng thức Bunnhiacopxki 

$2\left ( x^{2}+y^{2} \right )\geq \left ( x+y \right )^{2}\geq 1$

$x^{2}+y^{2}\geq \frac{1}{2}$

ta lại có 

$P=3\left ( x^{4}+y^{4}+x^{2}y^{2} \right )-2\left ( x^{2}+y^{2} \right )+1$

$\Rightarrow P=3\left [ \frac{2\left ( x^{4}+y^{4} \right )}{4}+\frac{x^{4}+y^{4}+2x^{2}y^{2}}{2} \right ]-2\left ( x^{2}+y^{2} \right )+1$

Theo bất đẳng thức bunhiacopxki ta có

$P\geq 3\left [ \frac{\left ( x^{2}+y^{2} \right )^{2}}{4}+\frac{\left ( x^{2}+y^{2} \right )^{2}}{2} \right ]-2\left ( x^{2}+y^{2} \right )+1$

$\Rightarrow P\geq \frac{9}{4}\left ( x^{2}+y^{2} \right )^{2}-2\left ( x^{2}+y^{2} \right )+1$

đặt $x^{2}+y^{2}=a\left ( a\geq \frac{1}{2} \right )$

ta sẽ tìm GTNN của $\frac{9}{4}a^{2}-2a+1$

ta có $\frac{9}{4}a^{2}-2a+1=2\left ( a^{2}-a+\frac{1}{4} \right )+\frac{a^{2}+2}{4}$

$=2\left ( a-\frac{1}{2} \right )^{2}+\frac{\left ( \frac{1}{2} \right )^{2}+2}{4}\geq \frac{9}{16}$

$\Rightarrow P\geq \frac{9}{16}$ 

Dấu bằng xảy ra khi $x=y=\frac{1}{2}$

 

Điểm 10.




#484923 $\boxed{\text{Chuyên Đề}}$ Bất đẳng thức - Cực trị

Đã gửi bởi nguyentrungphuc26041999 on 26-02-2014 - 19:54 trong Bất đẳng thức và cực trị

96/ Cho $0<x;y;z \leq \frac{3}{2}$. Chứng minh rằng $\sum \sqrt{x^2+\frac{1}{x^2}} \geq \frac{3}{2}.\sqrt{17}$

 

97/ Cho x; y; z > 0. Chứng minh $\sum \frac{2x}{x^6+y^4} \leq \sum \frac{1}{x^4}$

 

98/ Cho x; y; z > 0 và $\sum \frac{a^5}{b+c}=\frac{3}{2}$. Chứng minh $\sum ab^2 \leq 3$

 

99/ Cho $a;b;c>0$ và $a+b+c \leq \frac{3}{2}$. Chứng minh $\sum (a+\frac{1}{b})^3 \geq \frac{375}{8}$

 

100/ Cho x; y; z > 0. Chứng minh $\sum \frac{x^7}{x^2+y^2} \geq \frac{\sum x^5}{2}$

99 

ta có 

$\sum \left ( a+\frac{1}{b} \right )^{3}\geq \frac{\left ( a+\frac{1}{a}+b+\frac{1}{b}+c+\frac{1}{c} \right )^{3}}{9}$

ta tìm min $\sum \left ( a+\frac{1}{a} \right )$

áp dụng bất đẳng thức côsi và bunhia

$\sum \left ( a+\frac{1}{a} \right )= \sum \left ( a+\frac{1}{4a} \right )+\sum \frac{3}{4a}\geq 3+\frac{27}{4\left ( a+b+c \right )}\geq \frac{15}{2}$

$\Rightarrow \sum \left ( a+\frac{1}{b} \right )^{3}\geq \frac{\left ( \frac{15}{2} \right )^{3}}{9}= \frac{375}{8}$




#484920 $\boxed{\text{Chuyên Đề}}$ Bất đẳng thức - Cực trị

Đã gửi bởi nguyentrungphuc26041999 on 26-02-2014 - 19:34 trong Bất đẳng thức và cực trị

96/ Cho $0<x;y;z \leq \frac{3}{2}$. Chứng minh rằng $\sum \sqrt{x^2+\frac{1}{x^2}} \geq \frac{3}{2}.\sqrt{17}$

 

97/ Cho x; y; z > 0. Chứng minh $\sum \frac{2x}{x^6+y^4} \leq \sum \frac{1}{x^4}$

 

98/ Cho x; y; z > 0 và $\sum \frac{a^5}{b+c}=\frac{3}{2}$. Chứng minh $\sum ab^2 \leq 3$

 

99/ Cho $a;b;c>0$ và $a+b+c \leq \frac{3}{2}$. Chứng minh $\sum (a+\frac{1}{b})^3 \geq \frac{375}{8}$

 

100/ Cho x; y; z > 0. Chứng minh $\sum \frac{x^7}{x^2+y^2} \geq \frac{\sum x^5}{2}$

98 

áp dụng bất đẳng thức côsi

$\frac{a^{5}}{b+c}+\frac{a^{3}\left ( b+c \right )}{4}\geq a^{4}$

mà  dễ chứng minh  $\sum a^{3}\left ( b+c \right )\leq 2\sum a^{4}$

$\Rightarrow a^{4}+b^{4}+c^{4}\leq 3$

tiếp tục côsi 

$a^{4}+a^{4}+a^{4}+1\geq 4a^{3}$

thiết lập các bất đăng thức tương tự cộng lại ta có 

$a^{3}+b^{3}+c^{3}\leq 3$

97 $\sum \frac{2x}{x^{6}+y^{4}}\leq \frac{2x}{2x^{3}y^{2}}=\frac{1}{x^{2}y^{2}}$

đến đây thì dễ rồi




#483440 [Topic]Hỏi đáp về việc Vẽ Hình!

Đã gửi bởi nguyentrungphuc26041999 on 16-02-2014 - 14:25 trong Vẽ hình trên diễn đàn

1801064_1527455350812952_903986004_n.jpg

Thử luôn.

sao hình mình up lên nhỏ thế




#483426 $x^{3}+y^{3}+z^{3}+6\geq (x+y+z)^...

Đã gửi bởi nguyentrungphuc26041999 on 16-02-2014 - 12:50 trong Bất đẳng thức - Cực trị

ta có theo GT suy ra 

$x+y+z\geq 3\sqrt[3]{xyz}$

ta có 

$x^{3}+y^{3}+z^{3}\geq \frac{\left ( x+y+z \right )^{3}}{9}$

bất đẳng thức cần chứng minh 

$\left ( x+y+z \right )^{3}+54\geq 9\left ( x+y+z \right )^{2}$

đặt $x+y+z=a$ $\left ( a\geq 3 \right )$

cần chứng minh $a^{3}+54\geq 9a^{2}$

áp dụng bất đẳng thức cauchy

$\frac{a^{3}}{2}+\frac{a^{3}}{2}+54\geq 9a^{2}$

đoạn cuối này sai rồi




#483335 CMR: $$\frac{a^2}{a+bc}+\frac{b^...

Đã gửi bởi nguyentrungphuc26041999 on 15-02-2014 - 22:19 trong Bất đẳng thức và cực trị

Cho $a,b,c>0$ thỏa mãn $ab+bc+ca=abc$. CMR: $$\frac{a^2}{a+bc}+\frac{b^2}{b+ac}+\frac{c^2}{c+ab}\geq \frac{a+b+c}{4}$$

.

cách khác

mấy lần trước làm sai lần nhưng đã sửa,lần này phải đúng làn đầu

ta có theo bất đẳng thức BCS 

$\sum \frac{a^{2}}{a+bc}\geq \frac{\left ( a^{2}+b^{2}+c^{2} \right )^{2}}{a^{3}+b^{3}+c^{3}+abc\left ( a+b+c \right )}$

ta cần chứng minh $4\left ( a^{2}+b^{2}+c^{2} \right )^{2}\geq \left ( a^{3}+b^{3}+c^{3} \right )\left ( a+b+c \right )+abc\left ( a+b+c \right )$

$\Leftrightarrow 3\left ( a^{2}+b^{2}+c^{2} \right )^{2}+2\left ( \sum a^{2}b^{2} \right )\geq \sum a^{3}b+\sum ab^{3}+abc\left ( a+b+c \right )$

theo BĐT Cauchy ta có 

$\frac{a^{4}+a^{2}b^{2}}{2}\geq a^{3}b$

thiết lập các bất đăng thức tương tự như thế cộng lại ,kết hợp với bất đẳng thức cần chứng minh,ta được bất đẳng thức mới cần chứng minh là 

$2\left ( a^{4}+b^{4}+c^{4} \right )+7\left ( \sum a^{2}b^{2} \right )\geq \left ( a^{2}+b^{2}+c^{2} \right )\left ( ab+bc+ca \right )+\left ( ab+bc+ca \right )^{2}$

theo cauchy ta có 

 

$\left ( a^{2}+b^{2}+c^{2} \right )\left ( ab+bc+ca \right )\leq \left ( a^{2}+b^{2}+c^{2} \right )^{2}$

cộng lại ta có đpcm




#483333 CMR: $$\frac{a^2}{a+bc}+\frac{b^...

Đã gửi bởi nguyentrungphuc26041999 on 15-02-2014 - 22:13 trong Bất đẳng thức và cực trị

nhân a vào mẫu được $a^{2}+abc=a^{2}+ab+bc+ca=(a+b)(a+c)$

nhầm




#483329 CMR: $$\frac{a^2}{a+bc}+\frac{b^...

Đã gửi bởi nguyentrungphuc26041999 on 15-02-2014 - 22:01 trong Bất đẳng thức và cực trị

$\frac{a^{2}}{a+bc}=\frac{a^{3}}{(a+b)(a+c)}$

Áp dụng BĐT Cô si 3 số:

$\frac{a^{3}}{(a+b)(a+c)}+\frac{a+b}{4}+\frac{a+c}{4}\geq \frac{3a}{4}$

CMTT =>$\sum \frac{a^{3}}{(a+b)(a+c)}\geq \frac{a+b+c}{4}$ =>  đpcm

hình như bạn nhầm chỗ này




#483319 giúp mình bài bđt này với :)

Đã gửi bởi nguyentrungphuc26041999 on 15-02-2014 - 21:40 trong Bất đẳng thức và cực trị

cho a,b,c,d>0 thoả ab+bc+cd+da=1. Chứng minh: $\frac{a^{3}}{b+c+d}+\frac{b^{3}}{c+d+a}+\frac{c^{3}}{a+b+d}+\frac{d^{3}}{a+b+c}\geq \frac{1}{3}$. cảm ơn các anh chị nhiều

theo BCS dạng phân thức

$\sum \frac{a^{3}}{b+c+d}=\sum \frac{a^{4}}{ab+ac+ad}\geq \frac{\sum a^{2}}{3}\geq \frac{ab+bc+cd+da}{3}=\frac{1}{3}$




#483295 $\sum \frac{a}{b}\geq\frac{...

Đã gửi bởi nguyentrungphuc26041999 on 15-02-2014 - 20:58 trong Bất đẳng thức và cực trị

gõ nhầm latex,fix rồi

sao display nhỉ

bài này em làm rồi nên biến đổi ngây ngây 1 hồi thôi,sai thì thôi

fix đi fix lại 3,4 lần mà vẫn display,ai giúp em với




#483293 $\sum \frac{a}{b}\geq\frac{...

Đã gửi bởi nguyentrungphuc26041999 on 15-02-2014 - 20:53 trong Bất đẳng thức và cực trị



Cho $a,b,c\geq 0$ thỏa mãn $a^2+b^2+c^2=1$.CMR:

$\sum \frac{a}{b}\geq\frac{a+c}{b+c}+\frac{a+b}{a+c}+\frac{b+c}{a+b}$

em nghĩ không cần đến cái giả thiết đó đâu,

chuyển vế biến đổi tương đương ta được

$\frac{c\left ( a-b \right )}{b\left ( b+c \right )}+\frac{a\left ( b-c \right )}{c\left ( c+a \right )}+\frac{b\left ( c-a \right )}{a\left ( b+a \right )}\geq 0$

giả sử $b$ là số nằm giữa 2 số a$a$ và $b$ 

$\Rightarrow \left ( b-a \right )\left ( b-c \right )\leq 0$

để khỏi viết lại vì cái này dài quá,biến đổi

$b\left ( c-a \right )= -c\left ( a-b \right )-a\left ( b-c \right )$

bất đẳng thức tương đương

$a\left ( a-b \right )\left ( \frac{1}{b\left ( b+c \right )-\frac{1}{a\left ( a+b \right )} \right )+a\left ( b-c \right )\left ( \frac{1}{c\left ( c+a \right )}-\frac{1}{a\left ( a+b \right )} \right ) \geq 0$

$\Leftrightarrow \frac{c\left [ \left ( a-b \right )^{2}\left ( a+b \right )+b\left ( a-b \right )\left ( a-c \right ) \right ]}{ab\left ( a+b \right )\left ( b+c \right )}+\frac{\left ( b-c \right )\left ( a-c \right )\left ( a+c \right )+a\left ( b-c \right )^{2}}{c\left ( a+c \right )\left ( a+b \right )}\geq 0$

bất đẳng thức cuối đúng do $b$ nằm giữa $a$ và $c$




#483213 Trận 3 - Hình học

Đã gửi bởi nguyentrungphuc26041999 on 15-02-2014 - 12:57 trong Thi giải toán Marathon cấp THCS 2014

tuy không được tham gia trận 3 nhưng em cũng xin đóng góp cách làm của mình

$AM$ cắt cạnh $BE$ và $BC$ lần lượt tại $J$ và $K$ 

$BN$ cắt $AC$ tại $L$ ,$CP$ cắt $AB$ tại $T$

áp dụng định lý Mê- nê-la uýt liên tiếp, ta có 

xét $\bigtriangleup BEF$ có $A\in BF,M\in EF,J\in BE$

$\Rightarrow \frac{ME}{MF}.\frac{AF}{AB}.\frac{JB}{JE}=1\left ( 1 \right )$

xét $\bigtriangleup BEC$ có $A\in EC,J\in BE ,K\in BC$

$\Rightarrow \frac{JE}{JB}.\frac{KB}{KC}.\frac{AC}{AE}\left ( 2 \right )$

nhân vế theo vế của $\left ( 1 \right )$ và $\left ( 2 \right )$ ta có 

$\Rightarrow \frac{ME}{MF}.\frac{AF}{AB}.\frac{KB}{KC}.\frac{AC}{AE}=1$

tương tự ta cũng có 

$\frac{PD}{PE}.\frac{TA}{TB}.\frac{BC}{CD}.\frac{CE}{CA}=1$

$\frac{NF}{ND}.\frac{LC}{LA}.\frac{AB}{BF}.\frac{BD}{CB}=1$

nhân 3 hệ thức trên lại với nhau ta được 

$\frac{ME}{MF}.\frac{PD}{PE}.\frac{NF}{ND}.\frac{KB}{KC}.\frac{TA}{TB}.\frac{LC}{LA}.\frac{FA}{FB}.\frac{CE}{AE}.\frac{DB}{DC}=1\left ( * \right )$

áp dụng định lý Cê-va cho $\bigtriangleup ABC$ có $AD,BE,CF$ đồng quy và $\bigtriangleup DEF$ có $DM,FP,EN$ đồng quy,

$\frac{FA}{FB}.\frac{CE}{EA}.\frac{DB}{DC}=1$ và $\frac{ME}{MF}.\frac{PD}{PE}.\frac{NF}{ND}=1\left ( ** \right )$

từ $\left ( * \right )$ và $\left ( ** \right )$ suy ra $\frac{KB}{KC}.\frac{TA}{TB}.\frac{LC}{LC}=1$

theo định lý Cê-va đảo ta có $AM,BN,CP$ đồng quy (đpcm)

untitled.JPG  




#483201 [Topic]Hỏi đáp về việc Vẽ Hình!

Đã gửi bởi nguyentrungphuc26041999 on 15-02-2014 - 11:41 trong Vẽ hình trên diễn đàn

thử phátuntitled.JPG